LSAT and Law School Admissions Forum

Get expert LSAT preparation and law school admissions advice from PowerScore Test Preparation.

 Administrator
PowerScore Staff
  • PowerScore Staff
  • Posts: 8950
  • Joined: Feb 02, 2011
|
#23132
Complete Question Explanation

Flaw in the reasoning. The correct answer choice is (C)

The author of this stimulus concludes that the gas tax would raise $50 billion and therefore is a good way to deal with the budget deficit. This conclusion is based on the fact that it would raise $50 billion a year at current consumption rates. Furthermore, it would also reduce gas consumption and result in environmental gains and reduce our dependence on foreign oil. The fishy thing about this argument is that yes, the gas tax would raise $50 billion, assuming that current consumption rates stay constant. But it would also reduce consumption, which means the current consumption rates will not stay the same.

Answer choice (A) There is no indication from the stimulus that the data is irrelevant. The data is very relevant, since it goes to show how the gas tax would affect consumption.

Answer choice (B) There is no indication that current consumption figures are incorrect—the stimulus makes no mention of this at all.

Answer choice (C): This is the correct answer choice. The gas tax, in concluding that it would earn the government $50 billion a year, makes the incompatible assumptions that gas consumption would stay constant, but that it will also reduce gas consumption. This answer choice accurately and directly describes the inconsistency.

Answer choice (D) There is no causal argument in this stimulus, so this cannot be the correct answer choice.

Answer choice (E) While the stimulus may hint at appealing to conscience towards the end, this is not what is wrong with the argument. Furthermore, the argument as a whole approaches the problem logically—not relying on gut feelings and conscience, as this answer choice suggests.
 demk26
  • Posts: 23
  • Joined: May 03, 2020
|
#76746
Hi PS,

Can we infer that consumption rates would not stay the same because the stimulus states: "the resulting drop in the demand for gasoline" ?

Thank you!
 Adam Tyson
PowerScore Staff
  • PowerScore Staff
  • Posts: 5387
  • Joined: Apr 14, 2011
|
#76954
It depends on which of the author's assertions we might choose to believe, demk26! If we accept that there would be a resulting drop in demand, we would have to infer that consumption would go down, but if we accept that the gas tax would raise $50 billion, we would have to infer that consumption would remain constant. That's the problem - those claims, and the inferences we would have to draw from them, are mutually exclusive! We cannot infer either, because to infer one we must deny the other, and we have no way to choose which to accept and which to reject.
 pca99095
  • Posts: 3
  • Joined: Jul 21, 2020
|
#80078
I thought that the conclusion of this argument is "it seems a perfect way to deal with the federal budget deficit." I chose (a) because the facts that 'drop in the demand for gasoline would be ecologically sound' and keeping country from being too dependent on foreign oil producers" seem irrelevant to deal with federal budget deficit. How can they be relevant? I don't get it..
 Jeremy Press
PowerScore Staff
  • PowerScore Staff
  • Posts: 1000
  • Joined: Jun 12, 2017
|
#80115
Hi pca,

You've found the right conclusion, but there are a couple problems with your reading of answer choice A. To say in a blanket sense that the author cites irrelevant data is too strong, because the data cited in the author's primary premises (the first sentence and the first half of the second sentence) are extremely relevant to the conclusion. Citing irrelevant data isn't really problematic for an argument (other than making it longer than it needs to be), unless those data are the only data cited. The other problem with answer choice A is a rhetorical one. The author is not intending the last sentence to specifically support the idea of the gas tax being a good way to reduce the deficit (after all, the author thinks that conclusion has already been supported by the initial data cited). Rather, in that final sentence the author is offering additional reasons to support the general wisdom of simply implementing the gas tax. In other words, the final sentence is intended to get the reader on board with the gas tax's being a good idea (generally speaking). Sure, that's not the primary subject of the conclusion, but nothing stops an author from deliberately expanding the scope of the argument this way, and there's nothing flawed with an author's doing that. What you should look for instead is an answer like answer choice C, which shows why the author's claims (taken as a whole) are not consistent.

I hope this helps!
 concrottrox11@gmail.com
  • Posts: 29
  • Joined: Dec 07, 2021
|
#93031
Hi,

Im confused why the answer isnt D) ? Wouldnt the cause be federal budget deficit? Also, isn't the conclusion causal because it would be if federal budget-> tax of 50cents would raise 50 billion ?

What flaw in the book fallsunder "incompatible assumptions" so that I can study it? Thank you!
 Robert Carroll
PowerScore Staff
  • PowerScore Staff
  • Posts: 1819
  • Joined: Dec 06, 2013
|
#93052
concrottrox11,

If answer choice (D) is the answer, and the budget deficit is the cause, then what's the effect? This answer is talking about a reverse cause and effect situation. So the deficit is the cause, and the author erroneously thinks that the deficit is an effect instead? In your diagram, you don't even mention the budget deficit. So get straight which is the cause and which is the effect, and then locate where the author switches those. If you can't find that, it's a dispositive indicator that answer choice (D) is not correct.

The flaw here is in Lesson 7, page 26 of the PowerScore Course Book, under the heading "Internal Contradiction".

Robert Carroll
User avatar
 Hosseingold29
  • Posts: 5
  • Joined: Oct 29, 2022
|
#98292
there isnt any gap in your statement ?
bc of you said raising the cost will definitely render to decreasing the consumption , and I say there is no necessity ...
please explain , thanks :)
 Luke Haqq
PowerScore Staff
  • PowerScore Staff
  • Posts: 927
  • Joined: Apr 26, 2012
|
#98298
Hi Hosseingold29!

I wasn't sure which prior statement your comment was referring to, but I'd be happy to address the concern you mentioned about cost, consumption, and necessity in this question.

To begin, let's take the conclusion: "This tax would have the additional advantage that the resulting drop in the demand for gasoline would be ecologically sound and would keep our country from being too dependent on foreign oil producers." The author is specifically referring to a $0.50/gallon tax. The conclusion seems odd because the author both indicates that increasing the tax to this amount would generate even more revenue, but also indicates that increasing it to this amount would drop the demand (in which case raising the tax to that amount might not end up generating even more revenue). This flaw is stated in general terms in answer choice (C): "The author makes incompatible assumptions."

With regards to raising the cost having the necessary result of decreasing consumption, the author of the stimulus is direct in the final sentence: "This tax would have the additional advantage that the resulting drop in the demand for gasoline..." Accepting the author's statements as true, the author indicates that there is a "resulting drop in demand" from the tax. So the author's statements do include a necessary, causal connection between the tax and a resultant drop in demand.

Get the most out of your LSAT Prep Plus subscription.

Analyze and track your performance with our Testing and Analytics Package.